Form des Wassers im rotierenden Eimer

Ich muss zeigen, dass die Wasseroberfläche in einem Eimer, der sich mit konstanter Winkelgeschwindigkeit dreht, eine parabolische Form hat. Ich bin ziemlich verwirrt von diesem Problem, aber ich habe Folgendes getan:

F C F + F G R A v = U = M ( Ω × R ) M G z ^
Wo F C F ist die Zentrifugalkraft, F G R A v ist die Schwerkraft, U ist potentielle Energie, R := ( X , j , z ) . So U ( z ) = M G M Ω 2 z , somit
U ( z ) = G M z 1 2 Ω 2 z 2 + C
was eine Parabel ist.

Bei diesem Ansatz habe ich versucht, die Tatsache zu nutzen, dass die Oberfläche für äquipotential ist F C F + F G R A v . Aber anscheinend ist mein Ansatz nicht ganz stichhaltig. Bitte geben Sie einen Vorschlag zur Herangehensweise an dieses Problem, da ich keine andere Idee habe.

Fun Fact: Die Tatsache, dass ein rotierender Flüssigkeitseimer die Form einer Parabel annimmt, hat sehr reale Anwendungen Liquid Mirror .
Ich habe einmal mit einem Techniker zusammengearbeitet, der zuvor für ein Unternehmen gearbeitet hat, das Parabolreflektoren auf diese Weise unter Verwendung von Epoxid als Flüssigkeit herstellte. Die gehärtete Oberfläche wurde anschließend aus der Spinnvorrichtung entfernt, poliert und mit Aluminium beschichtet.
Ich denke, mein Ansatz war überhaupt nicht schlecht. Ich hatte einfach vergessen, dass es so ist U die orthogonal zur Oberfläche ist, aber nicht U selbst.
verwandte Frage hier
BTW, da ist ein Tippfehler in deiner Frage. Ω 2 R 2 2 statt gleich mit z .
@LRDPRDX R hier ist ( 0 , 0 , z ) Ich finde.
Verwandtes Video auf YouTube: Spinnendes Wasser – Jeff Regester

Antworten (4)

Betrachten Sie den folgenden zylindrischen Behälter mit Flüssigkeit, der sich gleichmäßig dreht ω :

Rotierende Flüssigkeitsoberfläche

Betrachten Sie ein unendlich kleines flüssiges Element D M in der Höhe H über dem Minimum der Parabel. Die darauf wirkenden Kräfte sind:

1) Schwerkraft:

G D M
2) die Zentripetalkraft:
D F C = ω 2 R D M
Betrachten Sie den Winkel a :
bräunen a = D H D R = F C G D M = ω 2 R D M G D M = ω 2 R G
Das bedeutet, dass:
ω 2 R D R G D H = 0
Integrieren Sie diese Differentialgleichung:
0 R ω 2 R D R = 0 H G D H
ω 2 R 2 2 = G H
H = ω 2 R 2 2 G

Dies ist eine quadratische Parabel.

Das Entscheidende, was hier verwendet wird, ist das D R ist normal zur Wasseroberfläche.

Ihre potentielle Energiefunktion U ( z ) zeigt überhaupt nicht, dass die Wasseroberfläche parabolisch ist. Was Sie finden müssen, ist die funktionelle Form der rotierenden Wasseroberfläche, dh die Oberflächenhöhe z als Funktion von R in Zylinderkoordinaten R Und z . (Aufgrund der Rotationssymmetrie ϕ ist nicht erforderlich.) Die Zentrifugalkraft ist

F C F = M ω 2 R
und die Gravitationskraft ist
F G R A v = M G
Die Wasseroberfläche ist orthogonal zur Richtung der resultierenden Kraft
F = F C F + F G R A v
Somit ist die Neigung der Wasseroberfläche
D z ( R ) D R = | F C F | | F G R A v | = M ω 2 R M G
Daraus erhalten wir durch Integration
z z 0 = 1 2 G ω 2 R 2
So erhalten wir tatsächlich eine parabolische Oberfläche im rotierenden Wasser im Eimer.

Warum kann das Bernoulli-Prinzip nicht angewendet werden? Äußerer Druck hebt sich auf. Der Druckunterschied von der Mitte zur Wand des Behälters beträgt also 2 g∆H, wobei ∆H die Höhenänderung der Oberfläche von der Mitte zur Wand ist. Dies entspricht der Geschwindigkeitsänderung nach außen, gegeben durch W^2 mal R^2, wobei w die Winkelgeschwindigkeit und R der Radius oder die x-Koordinate ist. Daraus ergibt sich ∆h = (w^2/2g)r^2

Dieses Problem hat verschiedene Lösungen. Eine davon ist die Anwendung des Prinzips der stationären Aktion. Es ist praktisch, Zylinderkoordinaten zu verwenden:

R = R ( R , ϕ , z ) .
Betrachten wir ein Wasservolumen, das sich wie ein ganzer Körper dreht, dh jedes kleine Wasservolumen (Klecks) hat die gleiche Winkelgeschwindigkeit Ω . Der nächste Schritt besteht darin, dieses Problem auf ein statisches Problem zu reduzieren. Um dies zu erreichen, sollten wir uns das Wasser aus dem Bezugssystem ansehen, das sich ebenfalls mit Winkelgeschwindigkeit dreht Ω (um die gleiche Achse wie das Wasservolumen). Aus dieser Sicht ist das Wasser in Ruhe. Dann wirken zwei Kräfte auf einen Wasserklumpen M : Gravitation M G und zentrifugal M Ω 2 R . Welche Potenziale haben diese Kräfte? Also sind sie
v G R = M G z , v C F = M Ω 2 R 2 2
Es liegt an Ihnen, zu überprüfen, ob die oben genannten Potentiale die richtigen Kräfte hervorrufen. Daher besitzt das Wasser nur potentielle Energie. Jetzt können wir die Aktion schreiben:
S = T 1 T 2 D T v Ö l D 3 R { ρ G z ρ Ω 2 R 2 2 } = C Ö N S T v Ö l L D 3 R ,
Wo L ist Lagrange oder Lagrange-Dichte. Aber natürlich müssen wir diese gesamte Wassermasse berücksichtigen M w wird konserviert bzw
v Ö l ρ D 3 R = M w = C Ö N S T .
In zylindrischen Koordinaten sind die obigen Integrale ( ρ = 1 )
2 π 0 R R D R 0 z ( R ) { . . . } D z = π 0 R { G z 2 R Ω 2 z R 3 } D R
Und
2 π 0 R R D R 0 z ( R ) D z = 2 π 0 R R z D R
Wir haben also ein Variationsproblem mit zusätzlicher Bedingung. Es ist einfach, dies mit der Methode der Lagrange-Multiplikatoren zu lösen. Notwendige Voraussetzung für S Extrema mit zusätzlicher Bedingung haben (ca M w ) ist die Existenz eines solchen Multiplikators λ Das
D D R G z R G z = 0 ,
Wo
G = G z 2 R Ω 2 z R 3 + λ R z .
Was gibt
2 G z R Ω 2 R 3 + λ R = 0
oder
z ( R ) = Ω 2 R 2 2 G λ .
Voila .

Ich bin ziemlich spät mit der Frage, aber ich glaube, ich kenne einen einfacheren Weg. Das kannst du ganz einfach mit Energieerhaltung lösen, die die kinetische Energie eines Kreises aus Wasser mit konstantem Radius ist M R 2 Ω 2 2 , in einem idealisierten System kann die Energie nirgendwo anders hingehen, als die potentielle Energie des Wassers zu erhöhen, also M G z = M R 2 Ω 2 2 oder z = R 2 Ω 2 2 G .

Ich verstehe deine Gleichung nicht ganz. Sie sollten eine Erklärung hinzufügen. Sie können nicht einfach eine Gleichung schreiben und denken, dass sie Sinn macht. Bitte.
@LRDPRDX Was verstehst du nicht? Es ist wirklich sehr einfach. Der Begriff der kinetischen Energie ist der einzige Teil, der nicht explizit angegeben ist, sondern nur ersetzt v = R Ω in die Moor-Standardgleichung für kinetische Energie. Sie verwenden dieselbe Gleichung, die Sie in Ihrer eigenen Antwort umständlicher gefunden haben. Es ist sehr seltsam, wenn Sie die Lagrange-Mechanik verwenden können, aber Probleme mit der Energieerhaltung haben. Du kannst nicht einfach sagen, dass du etwas nicht verstehst, und dann denken, dass ich weiß, womit du Probleme hast. Bitte.
Was ist zum Beispiel "ein Wasserkreis mit konstantem Radius"?